This discussion board is a place to discuss our Daily Challenges and the math and science
related to those challenges. Explanations are more than just a solution — they should
explain the steps and thinking strategies that you used to obtain the solution. Comments
should further the discussion of math and science.
When posting on Brilliant:
Use the emojis to react to an explanation, whether you're congratulating a job well done , or just really confused .
Ask specific questions about the challenge or the steps in somebody's explanation. Well-posed questions can add a lot to the discussion, but posting "I don't understand!" doesn't help anyone.
Try to contribute something new to the discussion, whether it is an extension, generalization or other idea related to the challenge.
Stay on topic — we're all here to learn more about math and science, not to hear about your favorite get-rich-quick scheme or current world events.
Markdown
Appears as
*italics* or _italics_
italics
**bold** or __bold__
bold
- bulleted - list
bulleted
list
1. numbered 2. list
numbered
list
Note: you must add a full line of space before and after lists for them to show up correctly
# I indented these lines
# 4 spaces, and now they show
# up as a code block.
print "hello world"
# I indented these lines
# 4 spaces, and now they show
# up as a code block.
print "hello world"
Math
Appears as
Remember to wrap math in \( ... \) or \[ ... \] to ensure proper formatting.
2 \times 3
2×3
2^{34}
234
a_{i-1}
ai−1
\frac{2}{3}
32
\sqrt{2}
2
\sum_{i=1}^3
∑i=13
\sin \theta
sinθ
\boxed{123}
123
Comments
This is quite a famous inequality, and there are several approaches that could be taken. Here is the simplest proof that I know of.
Define f(a,b,c)=ab2+bc2+ca2+abc.
WLOG, we may assume that a is the median of the 3, IE we either have b≤a≤c or b≥a≥c.
Step 1: We will show that f(a,b,c)≤f(a,b+c,0). This approach is known as Smoothing.
This follows by expanding both sides, and we want to compare ab2+bc2+ca2+abc≤a(b+c)2, which simplifies to
bc2+ca2≤abc+ac2
This is equivalent to
c(a−b)(a−c)≤0
From the assumption that a is the middle number, we know that a−b,a−c will have different signs (or be 0). Since c is non-negative, hence the entire product will be ≤0.
This is essentially how we approach @Krishna Sharma 's Case 1.
Step 2: We show that subject to a+b=3, we have f(a,b,0)≤4=f(1,2,0).
By AM-GM, we get
f(a,b,0)=ab2=4×a×2b×2b≤4(3a+2b+2b)3=4.
Hence, the result follows.
However, I do not know of an easy way to motivate the approach, and in particular step 1. Any thoughts or comments?
I remember seeing a stronger version of this inequality somewhere recently (probably on Math SE) which stated the following:
If a,b,c are non-negative reals, then the following inequality holds:
ab2+bc2+ca2+abc≤274(a+b+c)3
Our required inequality trivially follows from the stated inequality. However, proving the said stronger inequality seems to be harder than I expected. Let me see if I can think of a proof to that. For the time being, others are welcome to post their proof (if any) for the stated stronger inequality.
Note that the inequalities are equivalent. What happened was that we normalized the inequality, meaning that we made all of the terms have the same polynomial degree. To do so, we multiplied, where necessary, by a+b+c=3.
This is a standard approach, and one that I would often (though not always) recommend to use if the terms have different degrees.
You did not perform the Lagrangian properly. At the IMO, this solution will be scored 0/7.
1. You did not state the equations.
2. You did not state how to solve the equations.
3. You missed out the equality case of (1,2,0).
@Aman Rajput
–
You missed it because you didn't take care of the boundary condition. IE You didn't perform the Lagrangian properly.
E.g. What is the maximum of f(x)=x2 on the interval [−2,2]? Do you say that " f′=0⇒x=0 hence the maximum is f(0)=0? No, we still have to check the boundary points, where they need not satisfy f′=0 in order to be a maximum on the restricted domain.
@Calvin Lin
–
I know what you are trying to say . But what i know is that the min / max will be obtained using lagrangian even if we missed out boundary condition or other ordered pairs of equality
For example, if you ignore the boundary condition when calculating the max of f(x)=x2 on the closed interval [1,−1], then you would not get any answer.
This problem becomes quite a standard exercise when you use Lagrange multipliers. I think that the challenge is to prove the given inequality without using that which would be the reason for the problem being tagged under Algebra.
@Aman Rajput
–
What Prasun said is actually true. Lagrange Multiplier makes this problem too easy. It's like you're using a chainsaw to cut a carrot.
On the other hand, you need to show that the extremal point you've found is a global maximum point as opposed to an inflection point or a global minimum point.
Easy Math Editor
This discussion board is a place to discuss our Daily Challenges and the math and science related to those challenges. Explanations are more than just a solution — they should explain the steps and thinking strategies that you used to obtain the solution. Comments should further the discussion of math and science.
When posting on Brilliant:
*italics*
or_italics_
**bold**
or__bold__
paragraph 1
paragraph 2
[example link](https://brilliant.org)
> This is a quote
\(
...\)
or\[
...\]
to ensure proper formatting.2 \times 3
2^{34}
a_{i-1}
\frac{2}{3}
\sqrt{2}
\sum_{i=1}^3
\sin \theta
\boxed{123}
Comments
This is quite a famous inequality, and there are several approaches that could be taken. Here is the simplest proof that I know of.
Define f(a,b,c)=ab2+bc2+ca2+abc. WLOG, we may assume that a is the median of the 3, IE we either have b≤a≤c or b≥a≥c.
Step 1: We will show that f(a,b,c)≤f(a,b+c,0). This approach is known as Smoothing.
This follows by expanding both sides, and we want to compare ab2+bc2+ca2+abc≤a(b+c)2, which simplifies to
bc2+ca2≤abc+ac2
This is equivalent to
c(a−b)(a−c)≤0
From the assumption that a is the middle number, we know that a−b,a−c will have different signs (or be 0). Since c is non-negative, hence the entire product will be ≤0.
This is essentially how we approach @Krishna Sharma 's Case 1.
Step 2: We show that subject to a+b=3, we have f(a,b,0)≤4=f(1,2,0).
By AM-GM, we get
f(a,b,0)=ab2=4×a×2b×2b≤4(3a+2b+2b)3=4.
Hence, the result follows.
However, I do not know of an easy way to motivate the approach, and in particular step 1. Any thoughts or comments?
Log in to reply
Minor typo in the last step where you use AM-GM.
4×a×(2b)2≤4(3a+2b+2b)3=4
Log in to reply
Thanks fixed.
Sir, What is meant by IE ?
Log in to reply
It's just another way to write i.e. which means "that is".
This is just a comment, not a solution.
I remember seeing a stronger version of this inequality somewhere recently (probably on Math SE) which stated the following:
Our required inequality trivially follows from the stated inequality. However, proving the said stronger inequality seems to be harder than I expected. Let me see if I can think of a proof to that. For the time being, others are welcome to post their proof (if any) for the stated stronger inequality.
Log in to reply
Note that the inequalities are equivalent. What happened was that we normalized the inequality, meaning that we made all of the terms have the same polynomial degree. To do so, we multiplied, where necessary, by a+b+c=3.
This is a standard approach, and one that I would often (though not always) recommend to use if the terms have different degrees.
The weaker version that you saw, was most likely
ab2+bc2+ca2≤4⇔ab2+bc2+ca2≤274(a+b+c)3.
Is some info missing? Because substituting a=b=c=34 yields a value around 9.5
Log in to reply
Ooops, the condition should have been a+b+c=3.
Let f(a,b,c)=ab2+bc2+ca2+abc+k(a+b+c−3) (say this as equation (1)
Solving these four equations ∂a∂f=0 ∂b∂f=0 ∂c∂f=0 a+b+c=3
we get a=b=c=1,k=−4
substituting this back in equation 1 , we get fmax(a,b,c)=1+1+1+1−4(0)=4
Q.E.D
@Calvin Lin sir
Log in to reply
This demonstrates the point of this problem.
You did not perform the Lagrangian properly. At the IMO, this solution will be scored 0/7.
1. You did not state the equations.
2. You did not state how to solve the equations.
3. You missed out the equality case of (1,2,0).
Log in to reply
I didn't miss the case 1,2,0 doesnt satisfy the third equation ,i.e. , df/dc = 0
Log in to reply
E.g. What is the maximum of f(x)=x2 on the interval [−2,2]? Do you say that " f′=0⇒x=0 hence the maximum is f(0)=0? No, we still have to check the boundary points, where they need not satisfy f′=0 in order to be a maximum on the restricted domain.
Log in to reply
Log in to reply
For example, if you ignore the boundary condition when calculating the max of f(x)=x2 on the closed interval [1,−1], then you would not get any answer.
Log in to reply
ok i will keep in mind always to check at boundary
This problem becomes quite a standard exercise when you use Lagrange multipliers. I think that the challenge is to prove the given inequality without using that which would be the reason for the problem being tagged under Algebra.
Log in to reply
i dont think so ... :)
Log in to reply
On the other hand, you need to show that the extremal point you've found is a global maximum point as opposed to an inflection point or a global minimum point.
Why not assume a=b= c=1 and do it
Log in to reply
Myth: An expression attains it's maximum or minimum when all (some) of the variables are equal.